Đến nội dung

Hình ảnh

TOPIC: Các bài toán có nội dung hình học phẳng tuyển chọn


  • Please log in to reply
Chủ đề này có 110 trả lời

#21
HungNT

HungNT

    Thượng sĩ

  • Thành viên
  • 273 Bài viết

Bài 7

Chuẩn không nhỉ.Nếu mình cho các tỉ số là 1,9... hết thì đâu hẳn đã đúng

Ý là sao bạn, ta kết hợp 2 ĐK $\frac{AM}{MD}+\frac{BM}{ME}+\frac{CM}{MF}\geq 6$ và $\frac{MD}{AM}+\frac{ME}{BM}+\frac{MF}{CM}\geq \frac{3}{2}$ mà



#22
Bui Ba Anh

Bui Ba Anh

    Thiếu úy

  • Thành viên
  • 562 Bài viết

Bài 7: Giả sử $G$ là trọng tâm của tam giác với $3$ đường trung tuyến $AI,BK,CP$ tạo thành $6$ tam giác $BGI,CGI,CGK,AGK,AGP,BGP$

Giả sử $M$ nằm trong(hoặc trên cũng được  :closedeyes: ) tam giác $AGK$ theo định lý Van Oben(bài 6) ta có:

$\frac{AM}{MD}=\frac{AF}{FB}+\frac{AE}{EC}\leq \frac{AF}{PB}+\frac{AE}{KC}\leq 2$

Mặt khác $\frac{BM}{ME}=\frac{BF}{FA}+\frac{BD}{DC}\geq \frac{BF}{PA}+\frac{BD}{IC}\geq 2$

Dấu $=$ xảy ra khi $M$ trùng $G$ Q.E.D

Chú ý: Bài này mình đã bỏ qua 2 bước chứng minh cơ bản nhưng quan trọng nhé! Đó là $F$ nằm giữa $A$ và $B$, $E$ nằm giữa $A$ và $K$

A-Q:)

Hình gửi kèm

  • untitled (1).PNG

Bài viết đã được chỉnh sửa nội dung bởi Bui Ba Anh: 14-09-2014 - 20:44

NgọaLong

#23
Bui Ba Anh

Bui Ba Anh

    Thiếu úy

  • Thành viên
  • 562 Bài viết

Bài 11: (Kết bài này) Cho lục giác lồi $ABCDEF$ có $AB=BC,CD=DE,EF=FA$. Chứng minh rằng: $\frac{BC}{BE}+\frac{DE}{DA}+\frac{FA}{FC}\geq \frac{3}{2}$. Dấu bằng xảy ra khi nào?

Bài 12: Cho tứ giác $ABCD$. Chứng minh rằng: $Min \left \{ AB,BC,CD,DA \right \}\leq \frac{\sqrt{AC^{2}+BD^{2}}}{2}\leq Max\left \{ AB,BC,CD,DA \right \}$

P/S:Còn tồn bài 5  :(  :( 


NgọaLong

#24
Bui Ba Anh

Bui Ba Anh

    Thiếu úy

  • Thành viên
  • 562 Bài viết

untitled.PNG

Giải bài 5: Qua $A$ kẻ $AD$ song song $BC$ (D thuộc (O)) và hạ $DK$ vuông góc $BC$

Do tính đối xứng nên $D,A$ đối xứng nhau qua đường trung trực của $BC$

Từ $\widehat{BCA}\geq \widehat{ABC}+30^{\circ}=>2\widehat{BCA}-2\widehat{ABC}\geq 60^{\circ}=> \widehat{AOB}-\widehat{AOC}\geq 60^{\circ}=> \widehat{AOD}=\widehat{AOB}-\widehat{BOD}=\widehat{AOB}-\widehat{AOC}\geq 60^{\circ}$

Mà tam giác $AOD$ cân tại $O$ nên $AD\geq R$

Do đó $OH+R=OH+OC=OK+OC>KC=KH+HC=AD+HC\geq R+HC=>OH>HC=>\widehat{COH}<\widehat{OCH}$

Từ đó $2\widehat{CAB}+2\widehat{COH}<\widehat{BOC}+2\widehat{OCH}=180^{\circ}=>\widehat{CAB}+\widehat{COH}<90^{\circ}(Q.E.D)$

A-Q:)


Bài viết đã được chỉnh sửa nội dung bởi Bui Ba Anh: 14-09-2014 - 21:33

NgọaLong

#25
chardhdmovies

chardhdmovies

    Thiếu úy

  • Thành viên
  • 638 Bài viết

Bài 11: (Kết bài này) Cho lục giác lồi $ABCDEF$ có $AB=BC,CD=DE,EF=FA$. Chứng minh rằng: $\frac{BC}{BE}+\frac{DE}{DA}+\frac{FA}{FC}\geq \frac{3}{2}$. Dấu bằng xảy ra khi nào?

Capture.PNG

đặt $AC=x;CE=y;EA=z$

theo bđt $ptoleme$ cho tứ giác $ACEF$ ta có  $AC.EF+CE.AF\geq AE.CF\Rightarrow FA(x+y)\geq FCz\Rightarrow \frac{FA}{FC}\geq \frac{z}{x+y}$

tương tự thì ta có $\frac{DE}{DA}\geq \frac{y}{z+x};\frac{BC}{BE}\geq \frac{x}{y+z}$

do đó $\frac{BC}{BE}+\frac{DE}{DA}+\frac{FA}{FC}\geq \frac{x}{y+z}+\frac{y}{z+x}+\frac{z}{x+y}\geq \frac{3}{2}$

dấu bằng xảy ra khi $ABCDEF$ là lục giác đều

 

bđt chặt hơn cho bài trên là $\frac{BC}{BE}+\frac{DE}{DA}+\frac{FA}{FC}\geq \frac{3}{2}+\frac{(AC-CE)^2+(CE-AE)^2+(AE-AC)^2}{(AC+CE)^2+(CE+AE)^2+(AE+AC)^2}$

 

NTP


Bài viết đã được chỉnh sửa nội dung bởi chardhdmovies: 14-09-2014 - 21:27

                                                                                    chúng tôi là 3 người từ lớp 10 cá tính:NRC,NTP,A-Q


#26
chardhdmovies

chardhdmovies

    Thiếu úy

  • Thành viên
  • 638 Bài viết

bài 13 : cho $\Delta ABC$ có $\widehat{BAC}=105^0$,đường trung tuyến $BM$ và đường phân giác $CD$ cắt nhau tại $K$ sao cho $KB=KC$.Gọi H là hình chiếu của $A$ xuống $BC$.

CMR $HA=HB$

bài 14 :cho tứ giác $ABCD$ nội tiếp đường tròn $(O)$.Một đường thẳng qua $C$ cắt các tia đối của tia $AB,AD$ lần lượt tại $M,N$

CMR: $\frac{4S_{BCD}}{S_{AMN}}\leq (\frac{BD}{CD})^2$

 

NTP


Bài viết đã được chỉnh sửa nội dung bởi chardhdmovies: 14-09-2014 - 21:49

                                                                                    chúng tôi là 3 người từ lớp 10 cá tính:NRC,NTP,A-Q


#27
chardhdmovies

chardhdmovies

    Thiếu úy

  • Thành viên
  • 638 Bài viết

$\blacksquare$ bổ đề $1$:trong $\Delta ABC$ $\widehat{BAC}\geq 90^0\Leftrightarrow BC^2\geq AB^2+AC^2$

$\blacksquare$  bổ đề $2$:trong tứ giác $ABCD$ với $I;J$ là trung điểm $AC,BD$ thì $AB^2+BC^2+CD^2+DA^2=AC^2+BD^2+4IJ^2$

Capture.PNG

$*$ quay lại bài toán:

từ bổ đề $2$  ta có $AC^2+BD^2\leq AB^2+BC^2+CD^2+DA^2$

$\Leftrightarrow \frac{\sqrt{AC^2+BD^2}}{2}\leq \frac{\sqrt{AB^2+BC^2+CD^2+DA^2}}{2}$

                                       $\leq\frac{ \sqrt{4max\left \{ AB^2;BC^2;CD^2;DA^2 \right \}}}{2}=max\left \{ AB,BC,CD,DA \right \}$

đặt $m=min\left \{ AB,BC,CD,DA \right \}$

không mất tính tổng quát $\widehat{BAD}+\widehat{ADC}\geq 180^0;\widehat{BAD}\geq 90^0$

dựng hình bình hành $ABED$ thì $DE$ nằm giữa $DB,DC$

gọi $I;J$ là trung điểm $AC,BD$

trong tam giác $ACE$ có $IJ$ là đường trung bình nên $CE=2IJ$

có hai trường hợp xảy ra

$\blacklozenge$ trường hợp $1$ :$E$ nằm trong tứ giác $ABCD$.Trong hai góc $\widehat{AEB};\widehat{AED}$ có ít nhất một góc nhọn

không mất tính tổng quát giả sử $\widehat{AEB}\leq 90^0\Rightarrow \widehat{CEB}\geq 90^0$

theo bổ đề $1$ thì $BC^2\geq BE^2+CE^2\Leftrightarrow BC^2-4IJ^2\geq AD^2$

theo bổ đề $2$ thì $AC^2+BD^2=AB^2+AD^2+CD^2+(BC^2-4IJ^2)\geq AB^2+AD^2+CD^2+AD^2\geq 4m^2$

$\Rightarrow m\leq \frac{\sqrt{AC^2+BD^2}}{2}$

$\blacklozenge$ trường hợp $2$:$E$ nằm ngoài tứ giác $ABCD$ 

khi đí $CB$ nằm giữa $CD,CE$.Do đó $\widehat{BEC}\geq \widehat{BED}=\widehat{BAD}\geq 90^0$

khi đó chứng minh tương tự trường hợp $1$

do đó bài toán được chứng minh

 

NTP


Bài viết đã được chỉnh sửa nội dung bởi chardhdmovies: 15-09-2014 - 03:56

                                                                                    chúng tôi là 3 người từ lớp 10 cá tính:NRC,NTP,A-Q


#28
tuananh2000

tuananh2000

    Thượng sĩ

  • Thành viên
  • 218 Bài viết

bài 13 : cho $\Delta ABC$ có $\widehat{BAC}=105^0$,đường trung tuyến $BM$ và đường phân giác $CD$ cắt nhau tại $K$ sao cho $KB=KC$.Gọi H là hình chiếu của $A$ xuống $BC$.

CMR $HA=HB$

bài 14 :cho tứ giác $ABCD$ nội tiếp đường tròn $(O)$.Một đường thẳng qua $C$ cắt các tia đối của tia $AB,AD$ lần lượt tại $M,N$

CMR: $\frac{4S_{BCD}}{S_{AMN}}\leq (\frac{BD}{CD})^2$

 

NTP

Bài 13:$\bigtriangleup AHC$ có $MA=MC=MH\rightarrow \widehat{MHC}=\widehat{MCH}=2\widehat{BCK}$

Mà $BK=CK$ ($gt$) $\rightarrow \widehat{BCK}=\widehat{KBC}\rightarrow \widehat{MHC}=2\widehat{KBC}$($1$)

Ta có $\widehat{MHC}=\widehat{KBC}+\widehat{KMH}$($2$)

Từ ($1$) và ($2$)

$\rightarrow \bigtriangleup MHB$ cân tại $H$

$\rightarrow HM=HB$

Ta biện luận như sau : $Nếu HA>HB \rightarrow \widehat{ABH}>\widehat{BAH}\rightarrow \widehat{BAH}<45$ độ $\rightarrow \widehat{HAC}$$>$ 60 độ $\rightarrow \widehat{AMH}$$<60$ độ$\rightarrow AH <HM\rightarrow AH<HB$($VÔ LÍ$). Biện luận tt có $đpcm$

p/S: BÀI NÀY HÌNH NHƯ EM LÀM CẢ CHỤC LẦN R`  :luoi:  :luoi:


Bài viết đã được chỉnh sửa nội dung bởi tuananh2000: 14-09-2014 - 22:29

Live more - Be more  


#29
Bui Ba Anh

Bui Ba Anh

    Thiếu úy

  • Thành viên
  • 562 Bài viết

Như vậy là các bạn ChardHDmovies và Tuananh2000 đã solve hầu hết các bài,tiếp tục nhé!

Bài 15: Cho tam giác $ABC$, $D$ là trung điểm của $AB$ và $I$ trên cạnh $BC$ thỏa mãn $BI=2IC$. Chứng minh rằng nếu $\widehat{ADC}=\widehat{BAI}$ thì tam giác $ABC$ là tam giác vuông

Bài 16: (Định lý Stewart dạng đơn giản) Cho điểm $D$ nằm trên cạnh $BC$ của tam giác $ABC$, khi đó ta có:

$AB^{2}.CD+AC^{2}.BC=BC.(AD^{2}+BD.DC)$

(=> Chú ý: Định lý Stewart chuẩn dành cho vectơ là: Cho ba điểm $A,B,C$ thẳng hàng và điểm $D$. Chứng minh rằng

  $DA^{2}.\overline{BC}+DB^{2}.\overline{CA}+DC^{2}.\overline{AB}+\overline{BC}.\overline{CA}.\overline{AB}=0$ các bạn sẽ được học ở lớp 10)


NgọaLong

#30
HungNT

HungNT

    Thượng sĩ

  • Thành viên
  • 273 Bài viết

 

Bài 15: Cho tam giác $ABC$, $D$ là trung điểm của $AB$ và $I$ trên cạnh $BC$ thỏa mãn $BI=2IC$. Chứng minh rằng nếu $\widehat{ADC}=\widehat{BAI}$ thì tam giác $ABC$ là tam giác vuông

 

untitled.PNG

AI giao CD tại F, dựng $DK//AI\left ( K \in BC \right )$

Do D là trung điểm AB nên K là trung điểm BI $=>IK=IC=\frac{BI}{2}$

$=>$ I là trung điểm KC, mà DK // AI nên F là trung điểm CD

$\Delta DFA$ có $\angle DAF=\angle FDA=>FA=DF=\frac{DC}{2}$

$\Delta ADC$ có đường trung tuyến bằng nửa cạnh tương ứng nên là tg vuông =>ĐPCM


Bài viết đã được chỉnh sửa nội dung bởi HungNT: 15-09-2014 - 12:49


#31
chieckhantiennu

chieckhantiennu

    Thiếu úy

  • Thành viên
  • 621 Bài viết

Bài 16: (Định lý Stewart dạng đơn giản) Cho điểm $D$ nằm trên cạnh $BC$ của tam giác $ABC$, khi đó ta có:

$AB^{2}.CD+AC^{2}.BC=BC.(AD^{2}+BD.DC)$

 

Vẽ đường cao AH.

Ta có: $AB^2=AH^2+BH^2=AH^2+(BD+DH)^2;AC^2=AH^2+CH^2=AH^2+(CD-DH)^2;AD^2=AH^2+DH^2$

$\Rightarrow AB^2.DC+AC^2.BD-AD^2.BC$

$=[AH^2+(BD+DH)^2].DC+[AH^2-(CD-DH)^2].BD-(AH^2+DH^2).BC$

$=BD^2.DC+DH^2.DC+CD^2.BD+DH^2.BD-DH^2.BC$

$=BD.DC(BD+CD)=BD.DC.BC$

Hình gửi kèm

  • nguy.JPG

Đỗ Hoài Phương

Một số phận..

Facebook: https://www.facebook.com/phuong.july.969


#32
tuananh2000

tuananh2000

    Thượng sĩ

  • Thành viên
  • 218 Bài viết

Bài 17 : Cho hình bình hành $ABCD$ . Qua $A$ vẽ đường thẳng d cắt $BD;BC;DC$ tại $H;I;K$. Xác định vị trí của đường thẳng $d$ để $HA.HI.HK$ đạt $GTNN$

Bài 18 : Cho tam giác $ABC$ . Trên tia đối của các tia $BA;CB;AC$ lần lượt lấy các điểm $M;N;P$ sao cho $BM=AC;CN=AB;AP=BC$.CMR : $S_{ABP}.S_{BCM}.S_{CAN}=(S_{ABC})^{3}$

P/s: Mấy bài này thuộc kiến thức lớp 8 nhưng vẫn chưa có lời giải chính thức , mong mấy anh cùng giải  :namtay  :namtay


Live more - Be more  


#33
Bui Ba Anh

Bui Ba Anh

    Thiếu úy

  • Thành viên
  • 562 Bài viết

Bài 18: 

untitled.PNG

$\frac{S_{ABP}}{S_{ABC}}.\frac{S_{BCM}}{S_{ABC}}.\frac{S_{CAN}}{S_{ABC}}=\frac{PA}{AB}.\frac{BM}{AB}.\frac{CN}{BC}=1=>S_{ABP}.S_{BCM}.S_{CAN}=(S_{ABC})^{3}(Q.E.D)$

A-Q:)


NgọaLong

#34
duypro154

duypro154

    Trung sĩ

  • Thành viên
  • 133 Bài viết

Bài 19:Cho tam giac ABC vuông tại  đường phân giác trong BD =6cặn cm và 5AD=3DC.Tính BC? 

(thầy mình kêu là kẻ đường phụ)


Bài viết đã được chỉnh sửa nội dung bởi Mikhail Leptchinski: 09-02-2015 - 21:26


#35
tuananh2000

tuananh2000

    Thượng sĩ

  • Thành viên
  • 218 Bài viết

 

Cho tam giac ABC vuông tại  đường phân giác trong BD =6cặn cm và 5AD=3DC.Tính BC? 

(thầy mình kêu là kẻ đường phụ)

 

Mình vẫn chưa biết cách kẻ đường phụ , nhưng theo mình nghĩ chỉ cần áp dụng t/c p.g, PTG và dãy tỉ số bằng nhau là ra . Đây là cách giải của mình :

Theo gt có $\frac{AD}{DC}=\frac{AB}{BC}=\frac{3}{5} \rightarrow AB = \frac{3BC}{5}\Leftrightarrow AB^{2}=\frac{9BC^{2}}{25}$

Theo dãy tỉ số bằng nhau thì $\frac{AD}{3}=\frac{DC}{5}=\frac{AC}{8}\rightarrow AC=\frac{AD.8}{3}\Leftrightarrow AC^{2}=\frac{64.AD^{2}}{9}$

Áp dụng PTG thì $AB^{2}+AC^{2}=BC^{2} \Leftrightarrow \frac{9BC^{2}}{25}+\frac{64AD^{2}}{9}=BC^{2}\rightarrow \frac{8AD}{3}=\frac{4BC}{5}$

Thay tiếp vào tam giác DBA thì ta được $AB^{2}+AD^{2}=6 \rightarrow BC=\frac{2\sqrt{30}}{3}$


Live more - Be more  


#36
Bui Ba Anh

Bui Ba Anh

    Thiếu úy

  • Thành viên
  • 562 Bài viết

Bài 20: Cho tam giác $ABC$, $M$ là điểm bên trong tam giác. Chứng minh rằng $Min\left \{ MA,MB,MC \right \}+MA+MB+MC<AB+AC+BC$

(Chú ý: đây là 1 bài rất hay khi dùng kiến thức vectơ, được ghi trong cuốn Tài liệu chuyên toán 10, nhưng mình thấy vẫn có cách giải THCS hay nên các bạn giải theo hướng này)

Bài 21: (Đề thi HSG Toán Quốc tế IMO 2013,bài số 4)Cho tam giác nhọn $ABC$ có trực tâm $H$. Gọi $M,N$ là chân các đường cao kẻ từ đỉnh $B,C$ của tam giác. Gọi $D$ là điểm trên cạnh $BC$, gọi $\omega _1$ là đường tròn đi qua các điểm $C,D,M$ và $\omega _2$ là đường tròn đi qua các điểm $B,D,N$. $DQ,DP$ lần lượt là đường kính của các đường tròn $\omega _1,\omega _2$Chứng minh rằng $P,Q,H$ thẳng hàng.


Bài viết đã được chỉnh sửa nội dung bởi Bui Ba Anh: 16-09-2014 - 12:09

NgọaLong

#37
dogsteven

dogsteven

    Đại úy

  • Thành viên
  • 1567 Bài viết

Bài 21:

 

Gọi $K$ và $K'$ là giao điểm của $AD$ với $(\omega_1); (\omega_2)$

 

Theo phương tích điểm $A$: $AK.AD=AN.AB=AM.AC=AK'.AD$

 

Suy ra $K\equiv K'$ hay $K$ là giao $(\omega_1); (\omega_2)$ và $A,K, D$ thẳng hàng.

 

Dễ thấy $QP \bot AD$

 

Gọi $H'$ là giao của $QP$ với đường cao $AR$ ở đỉnh $A$ của $ABC$

 

Theo phương tích điểm $A$: $AH'.AR=AK.AD=AN.AQ=AH.AR$ nên $H\equiv H'$


Quyết tâm off dài dài cày hình, số, tổ, rời rạc.


#38
dogsteven

dogsteven

    Đại úy

  • Thành viên
  • 1567 Bài viết

Bài 21:

 

$D,E,F$ là trung điểm $BC, CA, AB$

 

Giả sử $M\in AFDC, BFEC$

 

$MB+MC < BF+FE+EC$  và $MA+MC< AF+FD+DC$

 

Cộng lại.


Quyết tâm off dài dài cày hình, số, tổ, rời rạc.


#39
HungNT

HungNT

    Thượng sĩ

  • Thành viên
  • 273 Bài viết

Xin đóng góp chút =]]

Bài 22:  Cho tam giác đều ABC, (O) là đường tròn ngoại tiếp tam giác ABC. Điểm M thay đổi, thuộc cung nhỏ AC của  đường tròn tâm (O) (M khác A và C). CM cắt AB tại E, AM cắt BC tại F. Qua A kẻ đường thẳng song song với BC cắt đường thẳng EF tại D. Chứng minh EF luôn đi qua điểm cố định D khi M thay đổi. (Đề thi lớp 10 THPT chuyên Bình Dương 2014-2015)



#40
Bui Ba Anh

Bui Ba Anh

    Thiếu úy

  • Thành viên
  • 562 Bài viết

Bài 23: Cho tam giác $ABC$ có $\widehat{B}=\widehat{C}=72^{\circ}$. Tính giá trị của biểu thức $\frac{BC}{AB-BC}$

Bài 24: (Đề thi toán quốc tế IMO 2013,câu 3) . Cho tam giác $ABC$. Gọi $A_1,B_1,C_1$ thứ tự là các tiếp điểm của đường tròn bàng tiếp tam giác với các cạnh $BC,CA,AB$. Chứng minh rằng nếu tâm đường tròn ngoại tiếp tam giác $A_1B_1C_1$ nằm trên đường tròn ngoại tiếp tam giác $ABC$ thì tam giác $ABC$ là tam giác vuông.


NgọaLong




0 người đang xem chủ đề

0 thành viên, 0 khách, 0 thành viên ẩn danh